Ensino SuperiorLimites de uma Função Tópico resolvido

Poste aqui problemas sobre assuntos estudados no Ensino Superior (exceto os cobrados em concursos públicos e escolas militares).

Moderador: [ Moderadores TTB ]

Avatar do usuário
Autor do Tópico
eureka
sênior
Mensagens: 23
Registrado em: Sáb 11 Out, 2008 18:43
Última visita: 22-01-20
Jan 2014 05 16:37

Limites de uma Função

Mensagem não lida por eureka »

Calcule [tex3]\lim_{x\rightarrow \infty }\left [ \frac{1}{\sqrt{n^{2}+1}} + \frac{1}{\sqrt{n^{2}+2}} + \frac{1}{\sqrt{n^{2} + 3} } + ... + \frac{1}{\sqrt{n^{2} + n}}\right][/tex3]

a) Não se pode calcular
b) [tex3]1[/tex3]
c) [tex3]0[/tex3]
d) [tex3]\infty[/tex3]
e) [tex3]2[/tex3]

Última edição: caju (Seg 20 Jan, 2020 16:49). Total de 5 vezes.
Razão: tex --> tex3



Avatar do usuário
Cardoso1979
6 - Doutor
Mensagens: 4008
Registrado em: Sex 05 Jan, 2018 19:45
Última visita: 04-04-23
Localização: Teresina- PI
Jan 2020 19 20:50

Re: Limites de uma Função

Mensagem não lida por Cardoso1979 »

Observe

Suponho que o limite seja: [tex3]\lim_{n\rightarrow \infty }\left [ \frac{1}{\sqrt{n^{2}+1}} + \frac{1}{\sqrt{n^{2}+2}} + \frac{1}{\sqrt{n^{2} + 3} } + ... + \frac{1}{\sqrt{n^{2} + n}}\right][/tex3]

Solução:

[tex3]\lim_{n\rightarrow \infty }\left [ \frac{1}{\sqrt{n^{2}+1}} + \frac{1}{\sqrt{n^{2}+2}} + \frac{1}{\sqrt{n^{2} + 3} } + ... + \frac{1}{\sqrt{n^{2} + n}}\right]=[/tex3]

[tex3]\lim_{n\rightarrow \infty }\left [ \frac{1}{\sqrt{n^2.\left(1+\frac{1}{n^2}\right)}} + \frac{1}{\sqrt{n^2.\left(1+\frac{2}{n^2}\right)}} + \frac{1}{\sqrt{n^{2}.\left(1+\frac{3}{n^2}\right)} } + ... + \frac{1}{\sqrt{n^{2}.\left(1+\frac{1}{n}\right)}}\right]=[/tex3]

[tex3]\lim_{n\rightarrow \infty }\left [ \frac{1}{n.\sqrt{\left(1+\frac{1}{n^2}\right)}} + \frac{1}{n.\sqrt{\left(1+\frac{2}{n^2}\right)}} + \frac{1}{n.\sqrt{\left(1+\frac{3}{n^2}\right)} } + ... + \frac{1}{n.\sqrt{\left(1+\frac{1}{n}\right)}}\right]=[/tex3]

Aplicando o limite tendendo para mais infinito, fica;

[tex3]=\frac{1}{∞.\sqrt{\left(1+\frac{1}{∞^2}\right)}}+\frac{1}{∞.\sqrt{\left(1+\frac{2}{∞^2}\right)}}+\frac{1}{∞.\sqrt{\left(1+\frac{3}{∞^2}\right)}}+...+\frac{1}{∞.\sqrt{\left(1+\frac{1}{∞}\right)}}=[/tex3]

[tex3]=\frac{1}{∞.\sqrt{\left(1+\frac{1}{∞}\right)}}+\frac{1}{∞.\sqrt{\left(1+\frac{2}{∞}\right)}}+\frac{1}{∞.\sqrt{\left(1+\frac{3}{∞}\right)}}+...+\frac{1}{∞.\sqrt{\left(1+\frac{1}{∞}\right)}}=[/tex3]

[tex3]=\frac{1}{∞.\sqrt{\left(1+0\right)}}+\frac{1}{∞.\sqrt{\left(1+0\right)}}+\frac{1}{∞.\sqrt{\left(1+0\right)}}+...+\frac{1}{∞.\sqrt{\left(1+0\right)}}=[/tex3]

[tex3]=\frac{1}{∞.1}+\frac{1}{∞.1}+\frac{1}{∞.1}+...+\frac{1}{∞.1}=[/tex3]

[tex3]=\frac{1}{∞}+\frac{1}{∞}+\frac{1}{∞}+...+\frac{1}{∞}=0+0+0+...+0=0[/tex3]


Portanto, [tex3]\lim_{n\rightarrow \infty }\left [ \frac{1}{\sqrt{n^{2}+1}} + \frac{1}{\sqrt{n^{2}+2}} + \frac{1}{\sqrt{n^{2} + 3} } + ... + \frac{1}{\sqrt{n^{2} + n}}\right]=0[/tex3] , alternativa c).


Nota

Agora se realmente for :
[tex3]\lim_{x\rightarrow \infty }\left [ \frac{1}{\sqrt{n^{2}+1}} + \frac{1}{\sqrt{n^{2}+2}} + \frac{1}{\sqrt{n^{2} + 3} } + ... + \frac{1}{\sqrt{n^{2} + n}}\right][/tex3] não se pode calcular.



Bons estudos!




Avatar do usuário
Autor do Tópico
eureka
sênior
Mensagens: 23
Registrado em: Sáb 11 Out, 2008 18:43
Última visita: 22-01-20
Jan 2020 20 14:21

Re: Limites de uma Função

Mensagem não lida por eureka »

Show, muito obrigado pela explicação. Abraços.



Avatar do usuário
Cardoso1979
6 - Doutor
Mensagens: 4008
Registrado em: Sex 05 Jan, 2018 19:45
Última visita: 04-04-23
Localização: Teresina- PI
Jan 2020 20 14:48

Re: Limites de uma Função

Mensagem não lida por Cardoso1979 »

eureka escreveu:
Seg 20 Jan, 2020 14:21
Show, muito obrigado pela explicação. Abraços.
Disponha 👍!



Avatar do usuário
Cardoso1979
6 - Doutor
Mensagens: 4008
Registrado em: Sex 05 Jan, 2018 19:45
Última visita: 04-04-23
Localização: Teresina- PI
Mai 2020 18 21:09

Re: Limites de uma Função

Mensagem não lida por Cardoso1979 »

Atenção!!!!

Desconsidere a resolução acima! Estava muito fácil para ser verdade( é o que dá ir com muita sede ao pote rsrs , fiquei sempre com uma pulga atrás da minha👂 , volta e meia retornava a esta questão porque tinha algo estranho na minha solução, não me contentava com a mesma, então não queria carregar esse peso na minha consciência ).É chato ter que dizer isso, peço até perdão por ter causado esse transtorno , espero que ninguém tenha se sentido prejudicado ou que eu tenha prejudicado alguém , mais ela não está correta!! Só estaria correta , se o limite dado não tivesse as "reticências".

Antes de resolver a mesma , eu irei postar algumas perguntas ( com soluções ) , e alguns teoremas que estão "relacionados" com esses tipos de questões ou que possam até mesmo esclarecer em outras questões parecidas como esta, vai depender muito do autor da pergunta ( o que se pede na questão, de acordo com as informações dadas ) , é por isso que é importante o leitor ter um conhecimento muito amplo para poder aplicar corretamente ( logo de cara ) um Teorema ou outra técnica( método ) para resolver a questão de imediato!

Teorema da média aritmética:

Seja [tex3](a_{n})[/tex3] uma sucessão convergente. Se [tex3]\lim_{n \rightarrow \infty}a_{n}=a[/tex3] então [tex3]\lim_{n \rightarrow \infty}\frac{a_{1}+a_{2}+a_{3}+...+a_{n}}{n}=a[/tex3]

Exemplo: Calcular [tex3]\lim_{n \rightarrow \infty}\frac{1+\sqrt{2}+\sqrt[3]{3}+\sqrt[4]{4}...+\sqrt[n]{n}}{n}[/tex3]

Solução:

Vamos calcular o limite do seu enésimo termo, vem;

[tex3]\lim_{n \rightarrow \infty}\sqrt[n]{n}=\lim_{n \rightarrow \infty}e^{ln \ (n)^{\frac{1}{n}}}=e^{\lim_{n \rightarrow \infty}\frac{ln \ (n)}{n}}[/tex3]

Por L'Hôspital ;

[tex3]=e^{\lim_{n \rightarrow \infty}\frac{1}{n}}=e^0=1[/tex3]

Logo, pelo teorema da média aritmética

[tex3]\lim_{n \rightarrow \infty}\frac{1+\sqrt{2}+\sqrt[3]{3}+\sqrt[4]{4}...+\sqrt[n]{n}}{n}=1.[/tex3]



Teorema da média geométrica:

Seja [tex3](a_{n})[/tex3] uma sucessão convergente. Se [tex3]\lim_{n \rightarrow \infty}a_{n}=a[/tex3] então [tex3]\lim_{n \rightarrow \infty}\sqrt[n]{a_{1}×a_{2}×a_{3}× \ ... \ ×a_{n}}=a[/tex3]

Exemplo: Calcular [tex3]\lim_{n \rightarrow \infty}\sqrt[n]{n!}[/tex3] .

Solução:

[tex3]\lim_{n \rightarrow \infty}\sqrt[n]{n!}=\lim_{n \rightarrow \infty}\sqrt[n]{(1)×(2)×(3)×...×(n) }[/tex3]

Podemos aplicar o teorema da média geométrica , segue que

[tex3]\lim_{n \rightarrow \infty}\sqrt[n]{n}=1[/tex3]



Critério de Stolz-Cesaro:

Sejam [tex3](a_{n})[/tex3] e [tex3](b_{n})[/tex3] duas sucessões tal que :

I. [tex3](b_{n})[/tex3] é monótona e [tex3]\lim_{n \rightarrow \infty}a_{n}=\lim_{n \rightarrow \infty}b_{n}=0[/tex3] ou

II. [tex3](b_{n})[/tex3] é monótona e [tex3]\lim_{n \rightarrow \infty}b_{n}=+∞[/tex3] , então [tex3]\lim_{n \rightarrow \infty}\frac{a_{n}}{b_{n}}=\lim_{n \rightarrow \infty}\frac{a_{n+1}-a_{n}}{b_{n+1}-b_{n}}[/tex3]

Exemplo: Calcular [tex3]\lim_{n \rightarrow \infty}\frac{\sqrt{1+1^2}+\sqrt{2+2^2}+...+\sqrt{n+n^2}}{n^2+2}[/tex3]

Solução:

[tex3]a_{n}=\sqrt{1+1^2}+\sqrt{2+2^2}+...+\sqrt{n+n^2}[/tex3] → [tex3]a_{n+1}=\sqrt{1+1^2}+\sqrt{2+2^2}+...+\sqrt{n+n^2}+\sqrt{n+1+(n+1)^2}[/tex3]

Por outro lado,

[tex3]b_{n}=n^2+2→b_{n+1}=(n+1)^2+2[/tex3]

Como [tex3](b_{n})[/tex3] é monótona crescente e [tex3]\lim_{n \rightarrow \infty}b_{n}=+∞[/tex3] , podemos aplicar o critério de Stolz-Cesaro, então:

[tex3]\lim_{n \rightarrow \infty}\frac{a_{n}}{b_{n}}=\lim_{n \rightarrow \infty}\frac{a_{n+1}-a_{n}}{b_{n+1}-b_{n}}=\lim_{n \rightarrow \infty}\frac{\sqrt{n+1+(n+1)^2}}{2n+1}=\lim_{n \rightarrow \infty}\frac{\sqrt{n^2+3n+2}}{2n+1}=[/tex3]

[tex3]=\lim_{n \rightarrow \infty}\frac{\frac{\sqrt{n^2+3n+2}}{n}}{\frac{2n+1}{n}}=[/tex3]

[tex3]=\lim_{n \rightarrow \infty}\frac{\sqrt{1+\frac{3}{n}+\frac{2}{n^2}}}{2+\frac{1}{n}}=\frac{1}{2}[/tex3]


Vamos a outras questões:

72.Calcule o limite abaixo interpretando-o como uma soma de Riemann em que os intervalos são divididos em n subintervalos de igual comprimento.

[tex3]\lim_{n \rightarrow \infty}\sum_{k=1}^{n}\frac{n}{n^2+k^2} \ ; \ [0,1][/tex3] .

Questão extraída do livro Anton Bivens David , volume 1 ; 10 EDIÇÃO. Exercício do capítulo 5 . Seção 5.6 questão 72.

Também encontrei uma questão igual a essa no livro de James Stewart volume 1 , 7 EDIÇÃO , capítulo 5 , seção 5.2 exercício 72( coincidiu com a numeração da questão do Anton ) , só que escrito de uma maneira "diferente" . Veja a pergunta :

72. Expresse o limite como uma integral definida.

[tex3]\lim_{n \rightarrow \infty}\frac{1}{n}\sum_{i=1}^{n}\frac{1}{1+\left(\frac{i}{n}\right)^2}[/tex3]

Obs.1 Nessa questão o autor não fornece nenhum intervalo , porém , geralmente o intervalo que devemos adotar é [ 0 , 1 ] , já vi questões que o autor dá como sugestão o intervalo [ 1 , 2 ] ( vi no livro do Louis Leithold volume 1 ) não para essa mesma questão , mais sim para outras questões , vai depender muito da questão, no entanto , tem questões que você poderá usar tanto o intervalo [ 0 , 1 ] como o intervalo [ 1 , 2 ] , como eu disse depende do autor.

Obs.2 De certa forma o autor até que facilitou a questão neste caso, ficou só no "jeito" para que você de imediato identifique a função que você irá usar para integrar.

Mais a frente postarei outra questão do livro Michael Spivak 2° EDIÇÃO capítulo 21 questão 9-( VI ) , lá o autor já apresenta esse mesmo limite utilizando reticências.

Chega de conversa e vamos a resolução.

Solução ( com explicação ):

Como o próprio enunciado nos diz, para resolver o exercício de alguma forma precisamos enxergar esse limite como se fosse uma integral, então vamos ver como fazer isso. Por definição, a integral é algo nessa forma

[tex3]\int\limits_{a}^{b}f(x)dx=\lim_{n \rightarrow \infty}\sum_{i=1}^{n}f(x_{i})∆x[/tex3]

onde n é o número de subintervalos, [tex3]x_{i}[/tex3] é um ponto arbitrário de um dos intervalos e ∆x é o tamanho de cada subintervalo.

Então vamos começar pegando o intervalo [ 0 , 1 ] e dividindo-o em n subintervalos de comprimento igual, sendo assim , os intervalos serão da forma : [tex3]\left[\frac{x-1}{n},\frac{x}{n}\right][/tex3] . Disto isso , o tamanho de cada subintervalo é de [tex3]\frac{x}{n}-\frac{x-1}{n}=\frac{1}{n}=∆x[/tex3] . Agora vamos escolher algum ponto de cada um desses intervalos, para isso vamos tomar [tex3]x_{i}=\frac{x}{n}.[/tex3] Agora juntando todas as informações, temos


[tex3]\lim_{n \rightarrow \infty}\sum_{x=1}^{∞}\frac{n}{n^2+x^2} =[/tex3]

[tex3]\lim_{n \rightarrow \infty}\sum_{x=1}^{∞}\frac{n}{n^2.\left(1+\frac{x^2}{n^2}\right)} =[/tex3]

[tex3]\sum_{x=1}^{∞}\frac{1}{\left(1+\frac{x^2}{n^2}\right)}.\frac{1}{n} =[/tex3]

[tex3]\sum_{x=1}^{∞}\frac{1}{\left(1+x^2_{i}\right)}.∆x = \int\limits_{0}^{1}\frac{1}{1+x^2}dx[/tex3]

Fizemos a correspondência, agora devemos resolver a integral, como se trata de uma integral já conhecida ( integral imediata ), logo

[tex3]\int\limits_{0}^{1}\frac{1}{1+x^2}dx = arc \ tg(1)-arc \ tg(0)=\frac{π}{4}[/tex3] .

Outra maneira( com explicação mais direta ):

[tex3]\frac{n}{n^2+k^2}=\frac{1}{1+\frac{k^2}{n^2}}.\frac{1}{n}[/tex3]

[tex3]\sum_{k=1}^{n}\frac{n}{n^2+k^2}=\sum_{k=1}^{n}f(x^*_{k})∆x[/tex3]

Onde [tex3]f(x)=\frac{1}{1+x^2}[/tex3] , [tex3]x^*_{k}=\frac{k}{n}[/tex3] e [tex3]∆x=\frac{1}{n}[/tex3] para 0 ≤ x ≤ 1. Assim,

[tex3]\lim_{n \rightarrow \infty}\sum_{k=1}^{n}\frac{n}{n^2+k^2}=\lim_{n \rightarrow \infty}\sum_{k=1}^{n}f(x^*_{k})∆x=
\int\limits_{0}^{1}\frac{1}{1+x^2}dx=\frac{π}{4}[/tex3]


Outra questão do livro do Anton volume 1 , 10 EDIÇÃO ( Exercício de revisão do capítulo 5 ).

30. Em cada parte, encontre o limite interpretando-o como o limite de uma soma de Riemann, na qual o intervalo [ 0 , 1 ] está dividido em n subintervalos de mesmo comprimento.

a) [tex3]\lim_{n \rightarrow \infty}\frac{\sqrt{1}+\sqrt{2}+\sqrt{3}+...+\sqrt{n}}{n^{\frac{3}{2}}}[/tex3]

Solução:

[tex3]\frac{1}{n}\sum_{k=1}^{n}\sqrt{\frac{k}{n}}=\sum_{k=1}^{n}f(x^*_{k})∆x[/tex3] , onde f ( x ) = √x , [tex3]x^*_{k}=\frac{k}{n}[/tex3] , e [tex3]∆x=\frac{1}{n}[/tex3] para 0 ≤ x ≤ 1. Assim,

[tex3]\lim_{n \rightarrow \infty}\frac{1}{n}\sum_{k=1}^{n}\sqrt{\frac{k}{n}}=\int\limits_{0}^{1}x^\frac{1}{2}dx=\frac{2}{3}[/tex3]


b) [tex3]\lim_{n \rightarrow \infty}\frac{1^4+2^4+3^4+...+n^4}{n^5}[/tex3]

Solução:

[tex3]\frac{1}{n}\sum_{k=1}^{n}\left(\frac{k}{n}\right)^4=\sum_{k=1}^{n}f(x^*_{k})∆x[/tex3] , onde f ( x ) = x⁴ , [tex3]x^*_{k}=\frac{k}{n}[/tex3] , e [tex3]∆x=\frac{1}{n}[/tex3] para 0 ≤ x ≤ 1. Assim,

[tex3]\lim_{n \rightarrow \infty}\frac{1}{n}\sum_{k=1}^{n}\left(\frac{k}{n}\right)^4=\int\limits_{0}^{1}x^4dx=\frac{1}{5}[/tex3]

c) [tex3]\lim_{n \rightarrow \infty}\frac{e^{\frac{1}{n}}+e^\frac{2}{n}+e^\frac{3}{n}+...+e^\frac{n}{n}}{n}[/tex3] ( essa questão é igualzinha que tem no livro do Michael Spivak 2 EDIÇÃO capítulo 21 questão 9 - ( I ), e também no livro de cálculo do George B. Thomas volume 1, 12 EDIÇÃO , capítulo 5 exercícios adicionais e avançados questão 22 ).

Solução:

[tex3]\sum_{k=1}^{n}\frac{e^{\frac{k}{n}}}{n}=\sum_{k=1}^{n}f(x^*_{k})∆x[/tex3] , onde [tex3]f(x)=e^{x}[/tex3] , [tex3]x^*_{k}=\frac{k}{n}[/tex3] , e [tex3]∆x=\frac{1}{n}[/tex3] para 0 ≤ x ≤ 1. Assim,

[tex3]\lim_{n \rightarrow \infty}\sum_{k=1}^{n}\frac{e^\frac{k}{n}}{n}=\lim_{n \rightarrow \infty}\sum_{k=1}^{n}f(x^*_{k})=\int\limits_{0}^{1}e^xdx=e-1[/tex3]


70. Calcule ( exercício de revisão do capítulo 5 James Stewart , volume 1 , 7 EDIÇÃO ):

[tex3]\lim_{n \rightarrow \infty}\frac{1}{n}\left[\left(\frac{1}{n}\right)^9+\left(\frac{2}{n}\right)^9+\left(\frac{3}{n}\right)^9+...+\left(\frac{n}{n}\right)^9\right][/tex3]

Uma solução:

[tex3]\lim_{n \rightarrow \infty}\frac{1-0}{n}\sum_{i=1}^{n}\left(\frac{i}{n}\right)^9=\int\limits_{0}^{1}x^9dx=\left[\frac{x^{10}}{10}\right]_{0}^{1}=\frac{1}{10}[/tex3]

Obs. Nessa questão o autor não forneceu o intervalo, porém o intervalo a ser considerado é [ 0 , 1 ] , já expliquei sobre isso no início ( acima ).

Vamos mais uma questão do mesmo livro ( capítulo 5 , seção 5.3 ).

70. Calcule limite abaixo , reconhecendo primeiro a soma como uma soma de Riemann para uma função definida em [ 0 , 1 ].

[tex3]\lim_{n \rightarrow \infty}\frac{1}{n}

\left(\sqrt{\frac{1}{n}}+\sqrt{\frac{2}{n}}+\sqrt{\frac{3}{n}}+...+\sqrt{\frac{n}{n}}\right)[/tex3]

Uma solução:

[tex3]\lim_{n \rightarrow \infty}\frac{1-0}{n}\sum_{i=1}^{n}\sqrt{\frac{i}{n}}=\int\limits_{0}^{1}\sqrt{x} \ dx=\left[\frac{2x^{\frac{3}{2}}}{3}\right]_{0}^{1}=\frac{2}{3}-0=\frac{2}{3}[/tex3]

Problemas quentes ( também do livro James Stewart volume 1 , 7 EDIÇÃO do capítulo 5 )

17. Calcule [tex3]\lim_{n \rightarrow \infty}\left(\frac{1}{\sqrt{n}\sqrt{n+1}}+\frac{1}{\sqrt{n}\sqrt{n+2}}+...+\frac{1}{\sqrt{n}\sqrt{n+n}}\right)[/tex3]

Solução:

[tex3]\lim_{n \rightarrow \infty}\frac{1}{n}\left(\sqrt{\frac{n}{n+1}}+\sqrt{\frac{n}{n+2}}+...+\sqrt{\frac{n}{n+n}}\right)=[/tex3]

[tex3]\lim_{n \rightarrow \infty}\frac{1}{n}\left(\frac{1}{\sqrt{1+\frac{1}{n}}}+\frac{1}{\sqrt{1+\frac{2}{n}}}+...+\frac{1}{\sqrt{1+1}}\right)=[/tex3]

[tex3]\lim_{n \rightarrow \infty}\frac{1}{n}\sum_{i=1}^{n}f\left(\frac{i}{n}\right)[/tex3]

Onde [tex3]f(x)=\frac{1}{\sqrt{1+x}}[/tex3] .

Assim,

[tex3]\int\limits_{0}^{1}\frac{1}{\sqrt{1+x}} \ dx=[2\sqrt{1+x}]_{0}^{1}=2(\sqrt{2}-1)[/tex3]


Agora vamos resolver duas questões do livro de cálculo do George B. Thomas volume 1 , 12 EDIÇÃO ( capítulo 5 , exercícios adicionais e avançados )

21. Determine o limite [tex3]\lim_{n \rightarrow \infty}\left(\frac{1}{n+1}+\frac{1}{n+2}+...+\frac{1}{2n}\right)[/tex3] .

Obs. Essa questão é igual a que está contida no livro de cálculo do Michael Spivak 2° EDIÇÃO capítulo 21 questão 9 - ( I I I ).

Basta tomar [tex3]f(x)=\frac{1}{1+x}[/tex3] , pois é a função que se "encaixa" com o limite , muitas vezes não é nada fácil encontrar uma função que se adeque ao limite e nem sempre é possível encontrar numa função adequada, aí você irá tentar resolver através de outros métodos.

Assim,

[tex3]\lim_{n \rightarrow \infty}\left(\frac{1}{n+1}+\frac{1}{n+2}+...+\frac{1}{2n}\right)=\int\limits_{0}^{1}\frac{1}{1+x}dx=ln \ 2[/tex3]

22. Determine o limite [tex3]\lim_{n \rightarrow \infty}\frac{1}{n}(e^\frac{1}{n}+e^\frac{2}{n}+...+e^\frac{n-1}{n}+e^\frac{n}{n})[/tex3] ( já resolvida acima! )



25. Seja f( x ) uma função contínua. Expresse [tex3]\lim_{n \rightarrow \infty}\frac{1}{n}\left[f\left(\frac{1}{n}\right)+f\left(\frac{2}{n}\right)+...+f\left(\frac{n}{n}\right)\right][/tex3] como uma integral definida.

Solução:

[tex3]\lim_{n \rightarrow \infty}\sum_{i=1}^{n}\left(\frac{i}{n}\right)\left(\frac{1}{n}\right)=\lim_{n \rightarrow \infty}\frac{1}{n}\left[f\left(\frac{1}{n}\right)+f\left(\frac{2}{n}\right)+...+f\left(\frac{n}{n}\right)\right]=\int\limits_{0}^{1}f(x)dx[/tex3]

Ué!? É só isso? Sim!

Vamos a questão do livro de cálculo do autor Michael Spivak , 2° EDIÇÃO capítulo 21.


9. Muitos limites de aparência impressionante podem ser facilmente calculados ( principalmente por quem os constrói ) , uma vez que na verdade eles são somas inferiores ou superiores disfarçados. Com a ajuda dessa observação, calcule cada um dos seguintes limites. ( Observação: a lista contém um elemento errante que pode ser calculado por considerações elementares ).

Obs. No final do livro onde tem a parte do gabarito o autor pede para utilizar partições de [ 0 , 1 ] em n partes iguais.

I - [tex3]\lim_{n \rightarrow \infty}\frac{\sqrt[n]{e}+\sqrt[n]{e^2}+...+\sqrt[n]{e^n}}{n}=\int\limits_{0}^{1}e^xdx=e-1(já \ resolvida \ acima!)[/tex3]

I I - [tex3]\lim_{n \rightarrow \infty}\frac{\sqrt[n]{e}+\sqrt[n]{e^2}+...+\sqrt[n]{e^{2n}}}{n}[/tex3]

Solução:

[tex3]\lim_{n \rightarrow \infty}\frac{\sqrt[n]{e}+\sqrt[n]{e^2}+...+\sqrt[n]{e^{2n}}}{n}=\int\limits_{0}^{2}e^xdx=e^2-1(segue \ o \ mesmo \ raciocínio \ da \ questão \ \ acima!)[/tex3]

Obs. O intervalo a se considerar aqui é [ 0 , 2 ] ( observe o limite )


I I I - [tex3]\lim_{n \rightarrow \infty}\left(\frac{1}{n+1}+...+\frac{1}{2n}\right)=\int\limits_{0}^{1}\frac{1}{1+x}dx=ln \ 2( também \ já \ resolvido \ acima!)[/tex3] .


IV - [tex3]\lim_{n \rightarrow \infty}\left(\frac{1}{n^2}+\frac{1}{(n+1)^2}+...+\frac{1}{(2n)^2}\right)[/tex3] .

Uma solução:

[tex3]\frac{1}{n^2}+\frac{1}{(n+1)^2}+...+\frac{1}{(2n)^2} ≤ n.\frac{1}{n^2} ≤ \frac{1}{n}[/tex3] .

Assim,

n → ∞

Logo, o resultado tende para zero ( 0 ).


V - [tex3]\lim_{n \rightarrow \infty}\left(\frac{n}{(n+1)^2}+\frac{n}{(n+2)^2}+...+\frac{n}{(n+n)^2}\right)[/tex3] .

Obs. Tem uma pergunta parecida com essa no livro do Louis Leithold , volume 1 , 3° EDIÇÃO capítulo 5 5.5.

34. Expresse como uma integral definida : [tex3]\lim_{n \rightarrow \infty}\sum_{i=1}^{n}\frac{n}{(i+n)^2}[/tex3] ( Sugestão : considerar a função f para qual [tex3]f(x)=\frac{1}{x^2}[/tex3] em [ 1 , 2 ] ).

Uma solução ( vou resolver de acordo com o gabarito do Spivak ):

Vamos tomar [tex3]f(x)=\frac{1}{(1+x)^2}[/tex3]

Assim,

[tex3]\int\limits_{0}^{1}\frac{1}{(1+x)^2}dx=\frac{1}{2}[/tex3]




VI - [tex3]\lim_{n \rightarrow \infty}\left(\frac{n}{n^2+1}+\frac{n}{n^2+2^2}+...+\frac{n}{n^2+n^2}\right)=\int\limits_{0}^{1}\frac{1}{1+x^2}dx=\frac{π}{4}( já \ resolvida \ acima!)[/tex3] .


Vamos então a resolução da questão postada pelo usuário eureka ( autor da pergunta deste fórum )

Calcule [tex3]\lim_{n\rightarrow \infty }\left [ \frac{1}{\sqrt{n^{2}+1}} + \frac{1}{\sqrt{n^{2}+2}} + \frac{1}{\sqrt{n^{2} + 3} } + ... + \frac{1}{\sqrt{n^{2} + n}}\right][/tex3] .

Uma solução:

Temos que

[tex3]\frac{1}{\sqrt{n^{2}+1}} + \frac{1}{\sqrt{n^{2}+2}} + \frac{1}{\sqrt{n^{2} + 3} } + ... + \frac{1}{\sqrt{n^{2} + n}} ≤ n.
\frac{1}{\sqrt{n^2+1}} ≤ \frac{1}{\sqrt{1+\frac{1}{n^2}}}[/tex3]

Assim,

n → ∞

Logo , o resultado tende para um ( 1 ) e NÃO para zero quando n tende a + ∞ , portanto alternativa b).

Obs. Infelizmente não consegui encaixar nenhuma função para aplicar a integral, não falei que tem certos limites que nem sempre sai facilmente ou resolve com integrais.

E mais uma vez , mil perdões pelo transtorno causado pelo meu equívoco!







Uuuuuuuuuuuufa!😅







Bons estudos!



Avatar do usuário
MateusQqMD
5 - Mestre
Mensagens: 2693
Registrado em: Qui 16 Ago, 2018 19:15
Última visita: 21-02-24
Localização: Fortaleza/CE
Mai 2020 19 02:10

Re: Limites de uma Função

Mensagem não lida por MateusQqMD »

Que bela solução, professor Cardoso1979!!

Muito obrigado pela participação do senhor aqui no fórum! :)


"Como sou pouco e sei pouco, faço o pouco que me cabe me dando por inteiro."

Avatar do usuário
Cardoso1979
6 - Doutor
Mensagens: 4008
Registrado em: Sex 05 Jan, 2018 19:45
Última visita: 04-04-23
Localização: Teresina- PI
Mai 2020 19 08:32

Re: Limites de uma Função

Mensagem não lida por Cardoso1979 »

MateusQqMD escreveu:
Ter 19 Mai, 2020 02:10
Que bela solução, professor Cardoso1979!!

Muito obrigado pela participação do senhor aqui no fórum! :)
Ôpa!! Disponha 👍🙌✌🏼




Responder
  • Tópicos Semelhantes
    Respostas
    Exibições
    Última msg

Voltar para “Ensino Superior”